Integral Series Interval Of Convergence

Calculus Level 4

What is the interval of convergence of the series formed by the following integral? lim n x 4 n + 2 x 2 + 1 d x \lim_{n\rightarrow\infty}\int\dfrac{x^{4n+2}}{x^2+1}\text{ }dx

Details and Assumptions \textbf{Details and Assumptions}

n n is an integer.

( 1 , 1 ) (-1,1) [ 1 , 1 ) [-1,1) ( , ) (-\infty,\infty) [ 1 , 1 ] [-1,1]

This section requires Javascript.
You are seeing this because something didn't load right. We suggest you, (a) try refreshing the page, (b) enabling javascript if it is disabled on your browser and, finally, (c) loading the non-javascript version of this page . We're sorry about the hassle.

4 solutions

Trevor B.
Feb 24, 2014

You can't evaluate this limit directly, so let's take a look at the behavior of the limit for some easy-to-work-with values of n n . Let n = 1. n=1. The integral is this.

x 6 x 2 + 1 d x = x 6 + x 4 x 2 + 1 x 4 x 2 + 1 d x = x 4 x 4 + x 2 x 2 + 1 + x 2 x 2 + 1 d x = x 4 x 2 + x 2 + 1 x 2 + 1 1 x 2 + 1 d x = x 4 x 2 + 1 1 x 2 + 1 d x = x 5 5 x 3 3 + x arctan x \begin{aligned} \int\dfrac{x^6}{x^2+1}\text{ }dx&=\int\dfrac{x^6+x^4}{x^2+1}-\dfrac{x^4}{x^2+1}\text{ }dx\\ &=\int x^4-\dfrac{x^4+x^2}{x^2+1}+\dfrac{x^2}{x^2+1}\text{ }dx\\ &=\int x^4-x^2+\dfrac{x^2+1}{x^2+1}-\dfrac{1}{x^2+1}\text{ }dx\\ &=\int x^4-x^2+1-\dfrac{1}{x^2+1}\text{ }dx\\ &=\dfrac{x^5}{5}-\dfrac{x^3}{3}+x-\arctan x^* \end{aligned}

\text{ }^* Note: there is no need to add a + C +C because adding a constant does not change the interval of convergence.

Notice the pattern in the coefficients of x . x. Now expand the integral again with n = 2. n=2.

x 10 x 2 + 1 d x = x 10 + x 8 x 2 + 1 x 8 + x 6 x 2 + 1 + x 6 + x 4 x 2 + 1 x 4 + x 2 x 2 + 1 + x 2 + 1 x 2 + 1 1 x 2 + 1 d x = x 8 x 6 + x 4 x 2 + 1 1 x 2 + 1 d x = x 9 9 x 7 7 + x 5 5 x 3 3 + x arctan x \begin{aligned} \int\dfrac{x^{10}}{x^2+1}\text{ }dx&=\int\dfrac{x^{10}+x^8}{x^2+1}-\dfrac{x^8+x^6}{x^2+1}+\dfrac{x^6+x^4}{x^2+1}-\dfrac{x^4+x^2}{x^2+1}+\dfrac{x^2+1}{x^2+1}-\dfrac{1}{x^2+1}\text{ }dx\\ &=\int x^8-x^6+x^4-x^2+1-\dfrac{1}{x^2+1}\text{ }dx\\ &=\dfrac{x^9}{9}-\dfrac{x^7}{7}+\dfrac{x^5}{5}-\dfrac{x^3}{3}+x-\arctan x \end{aligned}

So there is obviously a pattern in the coefficients of x . x. The pattern will continue as x x tends to . \infty. For now, let's ignore the arctan x -\arctan x term. Generalize the pattern as a series. x x 3 3 + x 5 5 x 7 7 + = lim n k = 0 n ( 1 ) k × x 2 k + 1 2 k + 1 x-\dfrac{x^3}{3}+\dfrac{x^5}{5}-\dfrac{x^7}{7}+\ldots=\lim_{n\rightarrow\infty}\sum_{k=0}^n\dfrac{(-1)^k\times x^{2k+1}}{2k+1} Because the summation never has any undefined terms in it, you can eliminate the limit and say this. x x 3 3 + x 5 5 x 7 7 + = k = 0 ( 1 ) k × x 2 k + 1 2 k + 1 x-\dfrac{x^3}{3}+\dfrac{x^5}{5}-\dfrac{x^7}{7}+\ldots=\sum_{k=0}^\infty\dfrac{(-1)^k\times x^{2k+1}}{2k+1}

This is the power series for arctan x . \arctan x. Reinserting the arctan x -\arctan x term shows that the integral is equal to 0. 0.

........................................................................................................................ \text{........................................................................................................................}

Some things to know before this section \textit{Some things to know before this section}

1 ) lim n a n 1)\text{ }\displaystyle\lim_{n\rightarrow\infty}a^n is nonconvergent for \textit{is nonconvergent for} a > 1 |a|>1 and convergent to \textit{and convergent to} 0 0 for \textit{for} a < 1. |a|<1.

2 ) + + 2)\text{ }\infty-\infty+\infty-\infty+\ldots is nonconvergent . \textit{is nonconvergent}.

So since the integral is 0 , 0, the answer is ( , ) (-\infty,\infty) right? No, you have to take into consideration the domain of the power series for arctan x . \arctan x. You need to take into account three ranges: ( , 1 ) , (-\infty,-1), [ 1 , 1 ] , [-1,1], and ( 1 , ) . (1,\infty). Look at the behavior of the power series for arctan x \arctan x is for each of these ranges. For ( , 1 ) (-\infty,-1) and ( 1 , ) (1,\infty) the magnitudes of the terms increase, so the series will eventually become + + , \infty-\infty+\infty-\infty+\ldots, which is nonconvergent. Now look at [ 1 , 1 ] . [-1,1]. The magnitude of the terms gets smaller as the sequence progresses, which is one condition for convergence. You can prove that the power series for arctan x \arctan x is convergent for all x [ 1 , 1 ] x\in[-1,1] with the ratio test.

So the first part of the result of the integral is convergent for x [ 1 , 1 ] . x\in[-1,1]. For the second part, we are looking for the values of x x for which arctan x \arctan x is defined, which is ( , ) . (-\infty,\infty). The intersection of these sets is the final answer. ( , ) [ 1 , 1 ] = [ 1 , 1 ] (-\infty,\infty)\cap[-1,1]=\boxed{[-1,1]} ........................................................................................................................ \text{........................................................................................................................} So after all that, the integral is... 0 ? 0? Think about it. Look at the function lim n x 4 n + 2 x 2 + 1 d x \displaystyle\lim_{n\rightarrow\infty}\int\dfrac{x^{4n+2}}{x^2+1}\text{ }dx You are finding a formula for finding the area under this curve. When x > 1 , |x|>1, the top part will be an infinite area, resulting in divergence. When x < 1 , |x|<1, it will be 0. 0. This results in a convergent area. Now let x = ± 1. x=\pm1. The numerator can be arranged like this. lim n ( ± 1 ) 4 n + 2 = lim n ( ( ± 1 ) 2 ) 2 n + 1 = lim n 1 2 n + 1 \lim_{n\rightarrow\infty}(\pm1)^{4n+2}=\lim_{n\rightarrow\infty}\left((\pm1)^2\right)^{2n+1}=\lim_{n\rightarrow\infty}1^{2n+1} This is an indeterminate exponent of the form 1 , 1^\infty, but it is equal to 1. 1. So the integral is defined and noninfinite at x = ± 1. x=\pm1.

How do you put bullets in a solution in Brilliant? I tried the description environment, but it wasn't rendering.

Trevor B. - 7 years, 3 months ago

Log in to reply

use * . For example,

a b c \begin{aligned} *a \\ *b \\ * c \\ \end{aligned}

produces

  • a
  • b
  • c

Calvin Lin Staff - 7 years, 3 months ago

Log in to reply

Is that in the description environment, math delimiters \ ( and \ ), or in normal text? I still can't get it to work for some reason.


Calvin Edit:

use * . For example,

a b c \begin{aligned} *a \\ *b \\ * c \\ \end{aligned}

produces

  • a
  • b
  • c

Trevor B. - 7 years, 3 months ago

Log in to reply

@Trevor B. I've pasted what I typed into your comment. You can edit it to see what it is like.

It is in the normal text environment. I had the code typed up in Latex just so that it will not be read and processed.

Calvin Lin Staff - 7 years, 3 months ago

Log in to reply

@Calvin Lin I see what I did, thank you. I was using an asterisk instead of putting it in align.

Trevor B. - 7 years, 3 months ago

I didn't understand the point where you said that adding a constant + C +C does not change the interval of convergence. I mean the limit to which the series converges would change. Is the question asking for what values of x x does the series converge? I think I am not able to understand the question properly.

Mridul Sachdeva - 7 years, 3 months ago

For x R [ 1 , 1 ] x \in \mathbb{R} - [-1,1] , the function clearly diverges, as the numerator increases at a much larger rate, & intuitively, it is not of any use, to divide it with a such smaller number (the denominator), as it appears when n n \to \infty . However, for x [ 1 , 1 ] x \in [-1,1] , we have: x 2 > x 4 n + 2 x^2 > x^{4n+2} , so the function converges to 0 0 , and hence it's integral is convergent.

Note that a summation of a function or sequence (basically integrals are also summations) is convergent, iff the function converges to 0 0 .

As a follow up, find the interval of convergence of lim n x 4 n + 2 x 2 + 1 d x \displaystyle \lim_{n \to \infty} \int \frac{x^{\frac{4}{n} + 2}}{x^2+1} dx .

x 2 x 2 + 1 = [ x tan 1 x ] a b \displaystyle \int \frac{x^2}{x^2 + 1} = \left[x - \tan^{-1} x\right]_a^b

Michael Tong - 7 years, 3 months ago

I think it's a glitch,that I entered my answer as [ 1 , 1 ] [-1,1] , but it now shows "Good Try", as if I had attempted the wrong option.

A Brilliant Member - 7 years, 3 months ago
Oliver Bel
Apr 8, 2014

lim n x 4 n + 2 x 2 + 1 d x = lim n r = 0 2 n ( 1 ) r x 2 r 1 1 + x 2 d x = x r = 0 ( 1 ) r x 2 r 2 r + 1 arctan x + C \begin{aligned} &\lim\limits_{n\mathop\to\infty}\int\frac{x^{4n+2}}{x^2+1}\,\mathrm{d}x\\ =&\lim\limits_{n\mathop\to\infty}\int\sum_{r\mathop=0}^{2n}(-1)^r x^{2r}-\frac{1}{1+x^2}\,\mathrm{d}x\\ =& x\sum_{r\mathop=0}^{\infty}(-1)^r\frac{ x^{2r}}{2r+1}-\arctan{x}+C \end{aligned}

If x 1 \displaystyle x\le 1 then { x 2 r 2 r + 1 } \left\{ \frac{x^{2r}}{2r+1} \right\} decreases monotonically to 0 0 , hence by the alternating series test, the series converges. If x > 1 \displaystyle x\gt 1 the limit of the summand does not exist, hence the series diverges.

Cody Johnson
Feb 24, 2014

1 1 + x 2 = 1 x 2 + x 4 x 6 + \frac{1}{1+x^2}=1-x^2+x^4-x^6+\dots . Plug in x = ± 1 x=\pm1 and you get 1 1 + 1 1 + 1-1+1-1+\dots . But, ( ± 1 ) 4 m + 2 = 1 (\pm1)^{4m+2}=1 , so x ± 1 x\neq\pm1 . The only interval that doesn't include these is ( 1 , 1 ) (-1,1) .

It's the indefinite integral. So that's not the series you want.

By the way, I clicked on [ 1 , 1 ] [-1,1] but the problem told me I tried [ 1 , 1 ) . [-1,1). Anyone else have a problem with this?

Patrick Corn - 7 years, 3 months ago

Log in to reply

We were told to update the answer to [ 1 , 1 ] [-1,1] . This issue likely arose when we were updating the various options.

Calvin Lin Staff - 7 years, 3 months ago

i did too......

Mandar Sohoni - 7 years, 3 months ago

0 pending reports

×

Problem Loading...

Note Loading...

Set Loading...